]Which statements are true? Check all that apply.

The equation |–x – 4| = 8 will have two solutions.
The equation 3.4|0.5x – 42.1| = –20.6 will have one solution.
The equation StartAbsoluteValue StartFraction one-half EndFraction x minus StartFraction 3 Over 4 EndFraction EndAbsoluteValue equals 0. = 0 will have no solutions.
The equation |2x – 10| = –20 will have two solutions.
The equation |0.5x – 0.75| + 4.6 = 0.25 will have no solutions.
The equation StartAbsoluteValue StartFraction 1 Over 8 EndFraction x minus 1. EndAbsoluteValue equals 5. = 5 will have infinitely many solutions.

Answers

Answer 1

The correct answers are:

1. The equation |-x -4| = 8 will have two solutions. (True)

5. The equation |0.5x – 0.75| + 4.6 = 0.25 will have no solutions. (True)

What is Equation?

An equation is a mathematical statement with an 'equal to =' symbol between two expressions that have equal values.

1. The equation |-x -4| = 8 will have two solutions. (True)

|-x - 4| = 8

-x -4 = ±8

-x -4 = 8 and -x -4 = -8

-x = 8 + 4 and -x = -8 + 4

-x = 12 and -x = -4

x = -12 and x = 4

Therefore, it has two solutions x ∈ {-12, 4}

2. The equation 3.4|0.5x - 42.1| = -20.6 will have one solution. (False)

Since the right side of the equation has a negative value therefore, according to rules of absolute value equations, it has no solution.

3. The equation |1/2x - 3/4| = 0 will have no solutions. (False)

|(1/2)x - 3/4| = 0

(1/2)x - 3/4 = ±0

Since ±0 is the same

(1/2)x = 3/4

x = 2X3/4

x = 3/2

Therefore, it has one solution x = 3/2

4. The equation |2x – 10| = –20 will have two solutions. (False)

Since the right side of the equation has a negative value therefore, according to rules of absolute value equations, it has no solution.

5. The equation |0.5x – 0.75| + 4.6 = 0.25 will have no solutions. (True)

|0.5x – 0.75| + 4.6 = 0.25

|0.5x – 0.75| = 0.25 - 4.6

|0.5x – 0.75| = -4.35

Since the right side of the equation has a negative value therefore, according to rules of absolute value equations, it has no solution.

6. The equation |(1/8)x - 1| = 5 will have infinitely many solutions. (False)

|(1/8)x - 1| = 5

(1/8)x - 1 = ±5

(1/8)x - 1 = 5 and (1/8)x - 1 = -5

(1/8)x = 5 + 1 and (1/8)x = -5 + 1

(1/8)x = 6 and (1/8)x = -4

x = 6X8 and x = -4X8

x = 48 and x = -32

Thus, it has two solutions x ∈ {48, -32}

Learn more about Equations from:

https://brainly.com/question/10413253

#SPJ1


Related Questions

f(x) = x + 6 , find the ordered pair when x = -2.

(-2,4)

(2,-4)

(-2,-4)

(2,4)

Answers

-------------------------------------------------------------------------------------------------------------

Answer:  [tex]\textsf{Option A, (-2, 4)}[/tex]

-------------------------------------------------------------------------------------------------------------

Given:  [tex]\textsf{f(x) = x + 6}[/tex]

Find:  [tex]\textsf{f(-2)}[/tex]

Solution:  We need to plug in the value -2 into every variable x that we have in the expression and simplify to get the solution.

Plug in the values

[tex]\textsf{f(x) = x + 6}[/tex][tex]\textsf{f(-2) = -2 + 6}[/tex]

Simplify

[tex]\textsf{f(-2) = -2 + 6}[/tex][tex]\textsf{f(-2) = 4}[/tex]

Therefore, the ordered pair that would match would have a x-coordinate of -2 and a y-coordinate of 4 producing (-2, 4) which matches option A.

Give ur answer in standard form.
3 x 10^4 ÷6 x 10^-4

Answers

Answer:

5.0×10⁷

Step-by-step explanation:

(3÷6)×(10⁴÷10^-4)

(0.5)×(10⁸)

5.0×10^-1×10⁸

5.0×10⁷

[tex] \frac{3 \times {10}^{4} }{6 \times {10}^{ - 4} } \\ \\ \frac{1 \times {10}^{4 - ( - 4)} }{2 } \\ \\ \frac{ 1 \times {10}^{4 + 4} }{2} \\ \\ \frac{1 \times {10}^{8} }{2} \\ \\ 0.5 \times {10}^{8} .[/tex]

The correlation in error terms that arises when the error terms at successive points in time are related is termed _____.

Answers

Answer:

auto correlation is the answer to the question

f(x)=(2x−3)(x+6)(5x+6)f, left parenthesis, x, right parenthesis, equals, left parenthesis, 2, x, minus, 3, right parenthesis, left parenthesis, x, plus, 6, right parenthesis, left parenthesis, 5, x, plus, 6, right parenthesis has zeros at x=-6x=−6x, equals, minus, 6, x=-\dfrac{6}{5}x=− 5 6 ​ x, equals, minus, start fraction, 6, divided by, 5, end fraction, and x=\dfrac{3}{2}x= 2 3 ​ x, equals, start fraction, 3, divided by, 2, end fraction. What is the sign of fff on the interval -6

Answers

Answer:

566

Step-by-step explanation:

I got it right..................

f(x)=x^2. which of these is g(x) ?

a. g(x)=(1/5x)^2
b. g(x)=5x^2
c.g(x)=(1/4x)^2
d.g(x)=1/5x^2

Answers

Answer:

a

Step-by-step explanation:

we know that g(x) means y therefore in this case let's suppose we have the coordinate (5;y) so we must find the corresponding y value

*DO THE TRIAL AND ERROR METHOD*

take any equation above and substitute the value of x of which is 5 and the corresponding value should be 1.

[tex]g(x) = (\frac{1}{5} x) {}^{2} [/tex]

Please help me out…

Polygon ____ and polygon ____ are similar to polygon 1.

Answers

Polygon 3 and 4 are similar to polygon 1.

Polygons will be called similar if they are exactly in the same shape, but can be in different sizes.

Here given, Some of the images are images of the polygon 1 from similarity transformations.

So we have to find the similar polygon from the given picture.

From picture it is clear that Polygon 2 is similar to polygon 1 as the size of polygon 2 has been decreased from polygon 1 but the shape of polygon 2 is the same as polygon 1 as we can their corresponding angles are also equal.

Also, polygon 3 is similar to polygon 1 as the size of polygon 3 has been decreased from polygon 1 but the shape of polygon 2 is the same as polygon 1 and reversed of polygon 1. So polygon 3 is of the same orientation as polygon 1 as we can their corresponding angles are also equal.

So from above, it is clear that Polygon 1 is similar to polygon 3 and 4 as they have the same shape but different sizes.

Therefore Polygon 3 and Polygon 4 are similar to polygon 1.

Learn more about similar polygon

here: https://brainly.com/question/1493409

#SPJ10

A sequence is defined by the recursive function f(n + 1) = one-half(n). If f(3) = 9 , what is f(1) ?

Answers

The value of f(1) is 81

What is Recursive function?

Recursive Function is a function that repeats or uses its own previous term to calculate subsequent terms and thus forms a sequence of terms

Given:

f(n + 1) = 1/3* f(n)

f(3) = 9

let n=0

f(0+1) = 1/3 * f(0)

f(1) = 1/3 * f(0)

Let n=1

f(1+1) = 1/3 * 1/3*f(0)

f(2) = 1/9 * f(0)

let n=2

f(2+1) = 1/3 * 1/9* f(0)

f(3) = 1/27 * f(0)

Also, f(3) =9

So,

9 = 1/27 * f(0)

f(0) = 27 * 9

f(0) = 243

Now,

f(1) = 1/3 * f(0)

f(1) = 1/3/ 243

f(1) = 81

Learn more about this concept here:

https://brainly.com/question/14216181

#SPJ1

Find the HCF of
x^4 y^2 and x^3 y^3

Answers

Here's your answer down here↓:

Step-by-step explanation:

x^3 - y^3 = (x - y)(x^2 + xy + y^2)

(x^4 - y^4) = (x^2 - y^2)(x^2 + y^2) = (x - y)(x + y)(x^2 + y^2)

Ok. So the factor (x-y) appears once in the top line and once in the second line. So we are going to take it the least amount of times.

So, the factor (x + y) appears in the top line zero times and in the second line one time so we will take it where it appears the least which is zero times so we are still at (x - y)

And it Same goes for the factors of (x^2 + y^2) and (x^2 + xy + y^2)


The sin (0) = - and lies in Quadrant III. Find the exact values of the sine
6'
and cosine of 20.

Answers

Answer:

Can you give exanation

I don't understand

The expression 55 + 14m - 2n + 3p has _________ terms.

1. 3
2. 4
3. 2
4. 5

Answers

The expression has 3 terms

Carolyn bought two brands of socks. She bought a total of 5 pairs. One brand cost $2 each and the other brand cost $3 each. She spent $13. Find the number of pairs each brand of socks she bought

Answers

Answer:

Step-by-step explanation:

Declaration

Let Brand A number of 2.00 socks= x

Let Brand B number of 3.00 socks = y

Equations

Number of socks : x + y = 5

Cost of socks: 2x + 3y = 13

Solution

Multiply top equation by 2

2(x+ y = 5)

2x + 2y = 10

Write the new equation under the cost of socks equation

2x + 3y = 13

2x + 2y = 10       Subtract.

y = 3

She spent 3 dollars on 1 pair of socks.

Now she needs to calculate the number of 2.00 dollar socks

2x + 3y = 13

2x + 3*3 = 13                Combine the left

2x + 9 = 13                   Subtract 9 from both sides.

2x +9-9 = 13-9             Combine

2x = 4                          Divide by 2

2x/2 = 4/2

x = 2

Answer

#2 dollar socks = 2

#3 dollar socks = 3

A basketball coach recorded how many shots each of the 30 players at tryouts made in a set of 6 attempts.

Answers

Answer:

Step-by-step explanation:

what is the question?

Answer:

Median = A

Mean = B

Step-by-step explanation:

Khan Academy

During a catered lunch, an average of 4 cups of tea are poured per minute. The lunch will last 2 hours. How many gallons of tea should the caterer bring if there are 16 cups in one gallon?

Answers

Answer:

30

Step-by-step explanation:

4 cups x 60 hours a min x 2 hours of lunch ÷ 16 cups per gallon = 30 gallons

The answer is 30 gallons.

Firstly, convert the hours to minutes so you know how many minutes you're working with. 2*60=120 minutes.

Then, multiply 120 with 4 to get the number of tea (in cups) that is poured. 120*4=480 cups.

Finally, divide 480 cups with 16 cups to find out the number of gallons poured. 480/16 equals 30 gallons.

Hope this helped.

Lillian went shopping for a new phone because of a sale. The price on the tag was $20, but Lillian paid $16 before tax. Find the percent discount.pls help

Answers

Answer:

20 % discount

Step-by-step explanation:

Lilian got 4 dollars off of the original price    (20 - 16 = 4 )

4 / 20  * 100% = 20% off

The percentage of discount that Lillian gets because of sale is 20%.

What is percentage?

Percentage is a measurement to find value of given number out of hundred.

Given that,

The price of phone on tag = $20.

Lillian paid for phone = $16.

To find the percent discount on phone,

First find amount of discount = 20 - 16 = $4.

The percentage of discount = (4 / 20) x 100 = 20 %

Lillian gets 20% discount on new mobile.

To learn more on Percentage:

https://brainly.com/question/24120406

#SPJ2

Given: l || m; ∠1 ∠3

Prove: p || q

Horizontal and parallel lines l and m are intersected by parallel lines p and q. At the intersection of lines l and p, the uppercase left angle is angle 1. At the intersection of lines q and l, the bottom right angle is angle 2. At the intersection of lines q and m, the uppercase left angle is angle 3.

Complete the missing parts of the paragraph proof.



We know that angle 1 is congruent to angle 3 and that line l is parallel to line m because
. We see that is congruent to by the alternate interior angles theorem. Therefore, angle 1 is congruent to angle 2 by the transitive property. So, we can conclude that lines p and q are parallel by the

.

Answers

The parts that are missing in the proof are:

It is given

∠2 ≅ ∠3

converse alternate exterior angles theorem

What is the Converse of Alternate Exterior Angles Theorem?

The theorem states that, if two exterior alternate angles are congruent, then the lines cut by the transversal are parallel.

∠1 ≅ ∠3 and l║m because we are: given

By the transitive property,

∠2 and ∠3 are alternate interior angles, therefore, they are congruent to each other by the alternate interior angles theorem.

Based on the converse alternate exterior angles theorem, lines p and q are proven to be parallel.

Therefore, the missing parts pf the paragraph proof are:

It is given∠2 ≅ ∠3converse alternate exterior angles theorem

Learn more about the converse alternate exterior angles theorem on:

https://brainly.com/question/17883766

#SPJ1

Answer:

it is given

angle 2

angle 3

converse alternate exterior angles theorem

Step-by-step explanation:

What is the sum of the arithmetic sequence 3, 9, 15..., if there are 24 terms? (5 points)

Answers

There's a fixed difference of 6 between terms (9 - 3 = 6, 15 - 9 = 6, and so on). The first term of the sequence is 3, so the [tex]n[/tex]-th term is

[tex]3 + 6(n-1) = 6n - 3[/tex]

If there are 24 terms in the sum, then the last term is 6×23 - 3 = 135.

Let [tex]S[/tex] be the sum,

[tex]S = 3 + 9 + 15 + \cdots + 123 + 129 + 135[/tex]

Reverse the order of terms:

[tex]S = 135 + 129 + 123 + \cdots + 15 + 9 + 3[/tex]

If we add up the terms in the same positions, we get twice [tex]S[/tex] on the left side, while on the right side we observe that each pair of terms will sum to 138.

[tex]S + S = (3 + 135) + (9 + 129) + (15 + 123) + \cdots + (135 + 3)[/tex]

[tex]2S = 138 + 138 + 138 + \cdots + 138[/tex]

and since there are 24 terms in the sum, the right side is the sum of 24 copies of 138. In other words,

[tex]2S = 24 \times 138[/tex]

and solving for [tex]S[/tex] gives

[tex]S = \dfrac{24\times138}2 = \boxed{1656}[/tex]

The vertex of this parabola is at (3-2). When the value is 4 the
y-value is 3. What is the coefficient of the squared expression in the
parabola's equation?
A. 7
B. 1
C. -1
D. 5

Answers

Answer:

D

Step-by-step explanation:

the equation of a parabola in vertex form is

y = a(x - h)² + k

where (h, k ) are the coordinates of the vertex and a is a multiplier

here (h, k ) = (3, - 2 ) , then

y = a(x - 3)² - 2

to find a substitute the point (4, 3 ) into the equation

3 = a(4 - 3)² - 2 ( add 2 to both sides )

5 = a × 1² = a

the coefficient of the squared expression is 5

f(X)= 4X^2 + 7X -3 g(X) = 6X^3 - 7X^2-5 Find (f + g) (x).

Answers

By using the binary operator of addition, the result of summing f(x) = 4 · x² + 7 · x - 3 and g(x) = 6 · x³ - 7 · x² - 5 is equal to (f + g)(x) = 6 · x³ - 3 · x² + 7 · x - 8.

How to apply operations between functions

Binary operators is a operator that connects two functions. There are five binary operators between two functions: (i) Addition, (ii) Subtraction, (iii) Multiplication, (iv) Division, (v) Composition.

In this question we must apply the addition between two quadratic functions. In addition, we know by algebra that the sum of a quadratic function and a cubic function is equal to a cubic function. Hence, the resulting expression is:

(f + g)(x) = f(x) + g(x)

(f + g)(x) = (4 · x² + 7 · x - 3) + (6 · x³ - 7 · x² - 5)

(f + g)(x) = 6 · x³ - 3 · x² + 7 · x - 8

By using the binary operator of addition, the result of summing f(x) = 4 · x² + 7 · x - 3 and g(x) = 6 · x³ - 7 · x² - 5 is equal to (f + g)(x) = 6 · x³ - 3 · x² + 7 · x - 8.

To learn more on functions: https://brainly.com/question/21145944

#SPJ1

PLEASE ANSWER ASAP!!!!!! WILL GIVE BRAINLIEST!!!

Answers

Answer ya so  1 is the answer

Step-by-step explanation:

Answer:

1

Step-by-step explanation:

Given expression:

[tex]\sf \left(\dfrac{3a^{-2}b^6}{2a^{-1}b^5} \right)^2[/tex]

To find the value of the expression when a = 3 and b = -2, substitute these values into the expression:

[tex]\implies \sf \left(\dfrac{3(3)^{-2}(-2)^6}{2(3)^{-1}(-2)^5} \right)^2[/tex]

[tex]\textsf{Apply exponent rule} \quad a^{-n}=\dfrac{1}{a^n}[/tex]

[tex]\sf \implies \left(\dfrac{3\left(\dfrac{1}{3^2}\right)(-2)^6}{2\left(\dfrac{1}{3^1} \right)(-2)^5} \right)^2[/tex]

[tex]\sf \implies \left(\dfrac{3\left(\dfrac{1}{9}\right)(-2)^6}{2\left(\dfrac{1}{3} \right)(-2)^5} \right)^2[/tex]

[tex]\sf \implies \left(\dfrac{\left(\dfrac{3}{9}\right)(-2)^6}{\left(\dfrac{2}{3} \right)(-2)^5} \right)^2[/tex]

[tex]\sf \implies \left(\dfrac{\left(\dfrac{1}{3}\right)(-2)^6}{\left(\dfrac{2}{3} \right)(-2)^5} \right)^2[/tex]

[tex]\textsf{Apply exponent rule} \quad (-a)^n=a^n,\:\: \textsf{ if }n \textsf{ is even}[/tex]

[tex]\textsf{Apply exponent rule} \quad (-a)^n=-a^n,\:\: \textsf{ if }n \textsf{ is odd}[/tex]

[tex]\sf \implies \left(\dfrac{\left(\dfrac{1}{3}\right) (2^6)}{\left(\dfrac{2}{3} \right) (-(2^5))} \right)^2[/tex]

[tex]\sf \implies \left(\dfrac{\left(\dfrac{1}{3}\right) (64)}{\left(\dfrac{2}{3} \right) (-32)} \right)^2[/tex]

[tex]\sf \implies \left(\dfrac{\left(\dfrac{1 \times 64}{3}\right)}{\left(\dfrac{2 \times -32}{3} \right)} \right)^2[/tex]

[tex]\sf \implies \left(\dfrac{\dfrac{64}{3}}{\dfrac{-64}{3}} \right)^2[/tex]

When dividing fractions, flip the second fraction and multiply it by the first:

[tex]\implies \sf \left( \dfrac{64}{3} \times \dfrac {3}{-64} \right)^2[/tex]

[tex]\implies \sf \left( \dfrac{64 \times 3}{3 \times (-64)}\right)^2[/tex]

[tex]\implies \sf \left( \dfrac{192}{-192}\right)^2[/tex]

[tex]\implies \sf \left(-1\right)^2[/tex]

[tex]\textsf{Apply exponent rule} \quad (-a)^n=a^n,\:\: \textsf{ if }n \textsf{ is even}[/tex]

[tex]\sf \implies 1^2=1[/tex]

if x + 3y = 25 write y in terms of x and also find the two solutions of this equation

Answers

Answer:

y= 25/3 - x/3

Step-by-step explanation:

y= 25/3 - x/3

What is a 10% margin increase on 1,810.69?

Answers

Answer:

1991.759

Step-by-step explanation:

1810.69×%10=181.069

1810.69+181.069=1991.759

(8.6x107)-(9.1x10-8)simplify

Answers

Answer:

  85,999,999.999 999 909

Step-by-step explanation:

The expression represents the difference of a relatively large number and one that is relatively small. That difference is approximately the value of the large number. The exact value requires 17 digits for its proper expression. Most calculators and spreadsheets cannot display this many digits.

Standard form

The numbers in standard form are ...

  86,000,000 = 8.6×10^7

  0.000000091 = 9.1×10^-8

Difference

Their difference is ...

  86,000,000 -0.000000091 = 85,999,999.999 999 909

In scientific notation, this is ...

  8.599 999 999 999 990 9×10^7

The quadrilateral below is formed from a parallelogram and an
isosceles triangle.
Calculate the size of angle VQU.

Answers

Answer:

VQU =58°

Step-by-step explanation:

angle QUV is 61 because of F angles (I don't know what they call them in ur country)

triangle QUV is an isosceles triangles so base angles are the same

61+61=122

180-122=58

1 in = 2.54 cm
how many millimeters are in 10.5 feet?
A.266.7 mm
B. 1,260 mm
C. 320.04 mm
D. 3,200.4 mm

Answers

Answer:

[tex]\fbox {D. 3,200.4 mm}[/tex]

Step-by-step explanation:

Given :

[ 1 inch = 2.54 centimeters ]

Unit conversions to keep in mind :

1 feet = 12 inches1 cm = 10 mm

Solving

10.5 feet10.5 x 12 inches126 inches126 x 2.54 cm320.04 cm320.04 x 10 mm3200.4 mm
The answer is D 3,200.4 mm I get that answer be equal ovulating the millimeters by 10.5 feet divided by the equal force of us in typical jetstream. Therefore 1 inches 2.54 cm. Then if I divide that by two, I get 3,200.4.

What is the equation of the line that passes through the point (5,-2) and has a
slope of -2/5

Answers

-------------------------------------------------------------------------------------------------------------

Answer:  [tex]\textsf{y = -2/5x}[/tex]

-------------------------------------------------------------------------------------------------------------

Given:  [tex]\textsf{Passes through (5, -2) and slope of -2/5}[/tex]

Find:  [tex]\textsf{The equation that follows the details provided}[/tex]

Solution: We first need to plug into the point-slope form and after simplifying, distributing, and solving for y we will complete our equation.

Plug in the values

[tex]\textsf{y - y}_1\textsf{ = m(x - x}_1\textsf{)}[/tex][tex]\textsf{y - (-2) = -2/5(x - 5)}[/tex]

Distribute and simplify

[tex]\textsf{y + 2 = -2/5(x - 5)}[/tex][tex]\textsf{y + 2 = (-2/5 * x) + (-2/5 * (-5))}[/tex][tex]\textsf{y + 2 = -2/5x + 2}[/tex]

Subtract 2 from both sides

[tex]\textsf{y + 2 - 2 = -2/5x + 2 - 2}[/tex][tex]\textsf{y = -2/5x + 2 - 2}[/tex][tex]\textsf{y = -2/5x}[/tex]

Therefore, the final equation that follows the information that was provided is y = -2/5x.

An architect is designing new housing structures for the primate section at the zoo. Her plan is shown below. Which animals will live in a building that is similar to the main primate house? A orangutans B chimpanzees C gibbons D gorillas

Answers

Answer:

A

Step-by-step explanation:

:p

4. Write the function of a linear equation that includes the
points (1,3) and (2,9).

Answers

Answer:

y = 6x - 3

Step-by-step explanation:

the equation of a line in slope- intercept form ( linear function ) is

y = mx + c ( m is the slope and c the y- intercept )

calculate m using the slope formula

m = [tex]\frac{y_{2}-y_{1} }{x_{2}-x_{1} }[/tex]

with (x₁, y₁ ) = (1, 3 ) and (x₂, y₂ ) = (2, 9 )

m = [tex]\frac{9-3}{2-1}[/tex] = [tex]\frac{6}{1}[/tex] = 6 , then

y = 6x + c ← is the partial equation

to find c substitute either of the 2 points into the partial equation

using (1, 3 ) , then

3 = 6 + c ⇒ c = 3 - 6 = - 3

y = 6x - 3 ← equation of linear function

Add.

(3+x3+3x2)+(2x3−2−4x2)



Express the answer in standard form.

Enter your answer in the box.

Answers

Step-by-step explanation:

(3+3+3*2)+(2*3-2-4*2)

The value of the addition of the two polynomials is 3x³ - x² + 1.

What is addition of polynomial?

The addition of polynomials involves combining like terms of two or more polynomials to create a new polynomial. A polynomial is an expression consisting of variables, coefficients, and exponents, connected by addition and subtraction operations.

To add the given polynomials:

(3 + x³ + 3x²) + (2x³ - 2 - 4x²)

First, let's group the like terms:

(x³ + 2x³) + (3x² - 4x²) + (3 - 2)

Combine the coefficients of the like terms:

3x³ + 3x² - 4x² + 3 - 2

Simplify the expression:

3x³ - x² + 1

Therefore, the sum of the given polynomials is: 3x³ - x² + 1.

Learn more on addition of polynomial here;

https://brainly.com/question/10078818

#SPJ2

A company making tires for bikes is concerned about the exact width of its cyclocross tires. The company has a lower specification limit of 22.8 millimeters and an upper specification limit of 23.1 millimeters. The standard deviation is 0.19 millimeters and the mean is 22.9 millimeters. What is the process capability index for the process? Note: Round your answer to 4 decimal places.

Answers

The process capability index for the process is 0.1754.

How to calculate the index?

The first sided specification limit will be:

= (Upper specification limit - mean)/(3 × standard deviation)

= (23.1 - 22.9)/(3 × 0.19)

= 0.2/0.57

= 0.3508

The second sided specification limit will be:

= (22.9 - 22.8)/(3 × 0.19)

= 0.1/0.57

= 0.1754

The process capability index for the process is 0.1754 wine it's the lower value.

Learn more about capability index on:

brainly.com/question/15734839

#SPJ12

Triangle AA'B'C' is the image of AABC under a dilation.

Answers

The scale factor is 3
Other Questions
what additional assessment and stabilization activities should be completed Select the correct answer.Which of the following is when an author substitutes a more mild or indirect word for a harsh or unpleasant word?A. oxymoronB. idiomC. euphemismD. paradox In the diagram, point D divides line segment AB in the ratio of 5:3. If line segment AC is vertical and line segment CD is horizontal, what are the coordinates of point C? Diagram shows a line segment with endpoints A(2, minus 6) and B(10, 2). Point D lies on this line segment. A dashed segment extends up from point A until point C and then extends horizontally to right until point D, forming a right triangle. A. (2, -1) B. (2, -3) C. (5, -3) D. (7, -1) Reset Next If 1 < a x , then the minimum value of [tex] \rm log_{a}(x) + log_{x}(x) [/tex] is ?PLEASE HELP!!!! You are betting on a game with an expected value of -$0.33. What does this mean? 2 inches tall on week 4 3 inches tall on week 6 4 inches tall on week 8 Which is the northernmost of the East Coast waterways? Under Jacksons spoils system, the political party of a new president could In this activity, you'll write an essay that argues against the point of view expressed in a reading of your choice. Using research and a fair approach, you'll sway readers to adopt your point of view instead.This writing assignment focuses on the following skills:- Critiquing arguments and backing up counterclaims- Conducting research and evaluating sources- Summarizing objectively- Using effective transitions What must a reader consider to understand an authors purpose for writing? choose three answers. Mrs. Johnson calls to tell you she has not received her new plan id card yet, but she needs to see a doctor. What can she expect to receive from the plan after the plan has received her enrollment form?. Which type of cloud resembles a puffy cotton ball?A. CirrusB. Cumulonimbus C. Cumulus D. Stratus It's very urgent plss answer A family originally bought a home for $654,030. Now, a few years later, the home is worth$457,821. What was the percent of decrease in its value? An email of inspiring a friend to join the debate club Question A right triangle is removed from a rectangle to create the shaded region shown below. Find the area of the shaded region. Be sure to include the correct unit in your answer. If necessary, refer to the list of geometry formulas. 2 cm 6 cm 8 cm 10 cm 0 00 cm cm Centimeters cm ? Question A right triangle is removed from a rectangle to create the shaded region shown below . Find the area of the shaded region? Find f g and g f, if they exist. f={(-4,-5),(0,3),(1,6)} and g={(-5,0),(3,-4),(6,1)}. Consider the steps for determining the quotient of 24 + x412X-4The quotient will beexpression.Complete the statements to choose a numerator and denominator to represent the quotient.The numerator of the simplified quotient isThe denominator of the simplified quotient is types of socialization Amber has been asked to identify the meter of "Sonnet 18. She is highlighting the syllables that should be stressed.And often is his gold complexion dimm'dWhich syllables should Amber highlight? Select three options.Andof--tenishisgoldcom-